You are on page 1of 31

Visit

FREETESTPAPER.com
for more papers

Website: freetestpaper.com
Facebook.com/freetestpaper
Twitter.com/freetestpaper
3

1. (a) Use the substitution u = 2 x − 1 to find ∫ ( x − 1) 2 x − 1 dx . [3]

(b) Find ∫ sin 3 x sin x dx . [2]

 1− x 
2. Given that f ( x ) = ln   , find f ( 0) , f ' ( 0) , f '' ( 0) and f ''' ( 0 ) . Hence, write down
 1 + cos x 

the first four non-zero terms in the Maclaurin’s series for f ( x ) . [6]

Use your result to deduce the first four non-zero terms in the Maclaurin’s series for
 1 + cos x 
ln  . [1]
 1+ x 

3. A tunnel is built to facilitate the transportation of goods by trains between Country X and
Country Y . Due to differences in the rail systems between the two countries, two types of
tracks are used – the international track with track gauge of 1435 mm and the narrow track
with track gauge of 1000 mm (in rail transport, track gauge is the spacing on a railway
track). It is known that the cross-section of the tunnel is a half ellipse with centre O and
width MN (see diagram). The maximum height of the tunnel is 2000 mm. To standardize
the volumes of the goods to be transported, the areas ABCD and EFGH are made equal.
Find the width of the tunnel MN , giving your answer to the nearest mm. [4]

D C

H G
2000 mm

M E A O B F N
1000 mm
1435 mm

[Turn over
4

4. A group of student councillors bought an 8 m long piece of tarpaulin canvas to build a


tentage for a school event. The canvas would extend diagonally at an angle of θ from the
ground to a height of 3 m, where it will then stretch horizontally to the school building (see
diagram for the cross-sectional view).
tentage

School
Building 3m

(i) Given that the total cross-sectional area covered by the canvas is A m 2 , show that
9
A = 24 − 9cosecθ + cot θ . [2]
2
(ii) Find, by differentiation, the largest possible value of A . [5]

2
5. The function f is defined by f : x ֏ ln ( 2 − x ) − 2, x ∈ ℝ, x ≠ 2.

(i) By considering the graph of y = f ( x ) , give a reason why f −1 does not exist. [2]

The function h is defined by h : x ֏ f ( x ) , x ≥ a, a ∈ ℤ.


(ii) Find the largest possible domain of h such that h −1 exists. [1]

(iii) Define h −1 in a similar form. [3]

(iv) Find the set of values of x which satisfies the equation hh −1 ( x ) = h −1h ( x ) . [2]

6. A curve C has parametric equations x = et + sin t , y = et − cos t .


(i) Describe the shape of C as t → −∞ . [2]

(ii) (
Find the Cartesian equation of the normal to C at the point P eθ + sin θ , eθ − cos θ , )
where θ > 0 , giving your answer in the form y = mx + c . [3]
The normal to C at P meets the y -axis at the point D , and the curve C meets the
positive x -axis at the point E that has integral coordinates.
(iii) Find the coordinates of D and E . [3]
(iv) Describe the locus of the mid-point of DE as θ varies. [2]
5

C
7.
B
D
k j
O E
i

F
The diagram shows a vehicle ramp OBCDEF with horizontal rectangular base ODEF and
vertical rectangular face OBCD . Taking the point O as the origin, the perpendicular unit
vectors i , j and k are parallel to the edges OF , OD and OB respectively. The lengths of
OF , OD and OB are 2h units, 3 units and h units respectively.


(i) Show that OC = 3 j + hk . [1]


(ii) The point P divides the segment BC in the ratio 2 :1 . Find OP in terms of h . [1]
(iii) A vector parallel to the normal of the plane BCEF is given as a i + b k . By the use
a
of a scalar product, find the value of . Hence find the Cartesian equation of the
b
plane BCEF in terms of h . [4]
(iv) Take h = 3 . Find the shortest distance from the point Q (1, 2, 2 ) to the plane OPF.
[4]

8. Mac has a 400 000 m 2 farm and on his farm, an area of 60 000 m 2 is covered in weeds in
June, and in September, the area increases to 69 500 m 2 . The growth of weeds is such that
the area covered in weeds increases at a monthly rate directly proportional to its area. At
the same time, Mac does weeding at a constant rate of 4 000 m 2 per month. Let the area of
the farm covered in weeds at time t (in months) be A m 2 .

(i) By considering a differential equation, show that A = α ekt + λ , where α , k and λ


are constants to be determined. [5]

(ii) The region covered in weeds is in the shape of a circle. Find the monthly rate at
which the radius of the region changes when the radius is 200 m . [2]

(iii) Mac understands that having some weeds on the farm can be beneficial. Find the
dA
monthly rate at which Mac needs to do weeding if = 0 in September. [2]
dt
dA
(iv) Comment on the significance of = 0 in the context of this question. [1]
dt

[Turn over
6

9. A researcher conducted a study on the radioisotope, Iodine-131 (I-131) which has a half-
life of 8 days (i.e., the amount of I-131 is halved every 8 days). He first introduced
1000 mg of I-131 in an empty Petri dish on Day 1 and tracked the amount of I-131 in the
dish.
(i) State the amount of I-131 in the dish at the end of 16 days. [1]

After every 16 days, i.e., on Day 17, Day 33, Day 49 etc., the researcher added 1000 mg of
I-131 to the dish.

(ii) Find the amount of I-131, to the nearest mg, in the dish immediately after 1000 mg
of I-131 was added on Day 49. [3]

(iii) Show that the amount of I-131 in the dish will never exceed 1334 mg . [2]

The researcher discovered that he accidentally used a different radioisotope, Iodine-125


(I-125) on Day 1, which has a half-life of 60 days instead. He checked that he had indeed
used the correct I-131 on other occasions.

(iv) Find the total amount of radioisotopes I-125 and I-131 in the dish on Day 121,
giving your answer correct to the nearest mg. [4]

10. (a) The equation z 3 − az 2 + 2az − 4i = 0 , where a is a constant, has a root i .


(i) Briefly explain why i* may not necessarily be a root of the equation. [1]
(ii) Show that a = 2 + i . [2]
(iii) Hence, find the remaining roots of the equation in exact form. [5]
π
(b) The complex number z satisfies the equations z * −1 + i = 2 and arg ( z − 2i ) = .
4
By considering z = x + iy , find z . [5]
7

11. (a)
y

x
O

The graph of y = f ( x ) is shown in the diagram above. It has asymptotes x = 1 and


y = 2 . The points A , B , C and D have coordinates ( 0, 1) , ( 2, 0 ) , ( 4, 3) and
( −1, 0 ) respectively, with C and D being stationary points.
On separate diagrams, sketch the graphs of
(i) y = f ( 2 x −1) , [3]
(ii) y = f ' ( x) , [3]
(iii) y = − f ( x). [3]
In each case, state the coordinates of A , B , C and D whenever applicable, and the
equations of any asymptotes.

2x + a
(b) A curve G with equation y = , where a and b are constants, has a stationary
x2 − b
 1
point at  −4, −  and a vertical asymptote x = −2 .
 4

(i) Find the values of a and b . [2]

(ii) Find the range of values of x for which G is increasing and is concave
downwards. [2]
(iii) By sketching a suitable line on the same diagram as G, find the number of
distinct real roots of the equation 5 x3 + 2 x 2 − 14 x + 7 = 0 . [3]

[Turn over
2015 H2 Mathematics C2 Prelim Paper 1 Solutions
Qn Solutions
1(a) Let u = 2 x − 1
1  u +1 
∫ ( x − 1) 2 x − 1 dx = 2 ∫  2 − 1 u du
1 u 1 
2 ∫ 2 2 
 + − 1 u du

1 32 1 2 5 2 3 
1
1
= ∫ ( u − 1) u du = ∫ u − u 2 du =  u 2 − u 2  + C
4 4 4 5 3 
1 52 1 32 1 5
1 3
= u − u + C = ( 2 x − 1) 2 − ( 2 x − 1) 2 + C
10 6 10 6
1(b) Using MF15
P+Q P −Q
= 3x =x
2 2
P + Q = 6x P − Q = 2x
2 P = 8 x => P = 4 x
Q = 2x
1
∴ ∫ sin 3 x sin x dx = ∫ − ( cos 4 x − cos 2 x ) dx
2
1  sin 4 x sin 2 x  sin 4 x sin 2 x
=−  −  +C = − + +C
2 4 2  8 4
2 f ( x ) = ln (1 − x ) − ln (1 + cos x ) , f ( 0 ) = − ln 2
−1 sin x
f '( x ) = + , f '(0) = −1
1 − x 1 + cos x

f ''( x) =
−1
+
(1 + cos x ) cos x − sin x ( − sin x )
2 2
(1 − x ) (1 + cos x )
−1 cos x + cos 2 x + sin 2 x
f ''( x ) = 2
+ 2
(1 − x ) (1 + cos x )
−1 1 1
f ''( x) = 2
+ , f '' ( 0 ) = −
(1 − x ) 1 + cos x 2
−2 sin x
f '''( x) = 3
+ 2
, f ''' ( 0 ) = −2
(1 − x ) (1 + cos x )
Using series formula in MF15:
x2 x3
f ( x ) = f ( 0) + xf ' ( 0) + f '' ( 0) + f ''' ( 0) + ...
2! 3!
1 2 1 3
f ( x ) = − ln 2 − x − x − x + ...
4 3
 1 + cos x   1− ( −x)  1 2 1 3
ln   = − ln   = ln 2 − x + x − x + ...
 1+ x   1 + cos ( − x )  4 3
3 Let FG = h1 and let BC = h2 .
x2 y2
Form equation of the ellipse: 2 + = 1 --- (1)
a 20002
Since the areas of ABCD and EFGH are equal: 1000h2 = 1435h1 --------- (2)
 1435 
Substitute the point G  , h1  into (1):
 2 
2 2
717.5 h
2
+ 1 2 = 1 --------- (3)
a 2000
 1000 
Substitute the point C  , h2  into (1):
 2 
2 2
500 h
2
+ 2 2 =1 --------- (4)
a 2000
2
5002 (1.435h1 )
Substitute (2) into (4): + = 1 --- (5)
a2 20002
From (3) and (5): a 2 = 764806.25 ⇒ a = 874.532 ⇒ MN = 2a = 1749.06 ≈ 1749 mm
4(i) 3
sloped length =
sin θ
 3  1 3  9
total area =  8 −  ( 3) +   ( 3) = 24 − 9 cosec θ + cot θ
 sin θ  2  tan θ  2
4(ii) 9
A = 24 − 9 cosec θ + cot θ
2
dA 9
= 9 cot θ cosec θ − cosec 2 θ
dθ 2
9 2 cos θ − 1
= ⋅
2 ( sin θ ) 2
dA
Let = 0 , 2 cos θ = 1 .

π
So, θ = .
3
π 9 π 18 9 9 3
A = 24 − 9 cosec + cot = 24 − + = 24 − = 16.2 ( 3 s.f.)
3 2 3 3 2 3 2

π 
− +
π  π 
θ      
3 3 3
dA
+ 0 –

The maximum area is 16.2 m2.


5(i) y

y=2
x

x=2
Since a line such as y = 2 intersects the curve y = f ( x ) at 2 points, f is not one-to-
one. ∴ f −1 does not exist.
5(ii) Largest possible domain for h is [3, ∞ ) .
y+2
5(iii) 2
y = ln ( 2 − x ) − 2 ⇒ x = 2 ± e 2

x+2
−1 2
Since x ≥ 3, h : x → 2 + e , x ∈ ℝ, x ≥ −2
5(iv) −1 −1
hh ( x) = h h ( x) = x which is always valid since the inverse function exists.

Since domain of hh −1 is domain of Dh −1 = [ −2, ∞ ) and domain of h −1h is domain of


Dh = [3, ∞ ) . Thus the set of values is { x ∈ ℝ : x ≥ 3} .
6(i) As t → −∞ , x → sin t and y → − cos t .
Using trigonometric identity sin 2 t + cos 2 t = 1 , the Cartesian equation of C is
x2 + y 2 = 1. Thus the shape of C is a circle with centre at the origin and with unit
length radius.
6(ii) dx t dy t dy et + sin t
= e + cos t , = e + sin t . ∴ =
dt dt dx et + cos t
eθ + cos θ
At P, gradient of normal is − θ .
e + sin θ
eθ + cos θ
θ
(
∴ equation of normal is y − e − cos θ = − θ
e + sin θ
) ( (
x − eθ + sin θ ))
θ
e + cosθ eθ + cos θ
⇒ y=− θ
e + sin θ
( ( )) (
x − eθ + sin θ + eθ − cos θ ⇒ y = − θ )
e + sin θ
x + 2eθ
6(iii) Using equation of normal found in (ii), point D is ( 0, 2eθ ) .
E is a point on C.
From y = et − cos t , when y = 0 , ⇒ et = cos t
By inspection, t = 0 . ∴ x = e0 + sin 0 = 1 . Hence point E is (1, 0 ) .
6(iv) 1  1
The mid-point of DE is  , eθ  . Since x = is a fixed value and
2  2
1
θ > 0 ⇒ y = eθ > 1 ,∴ required locus is a half-line x = , with y > 1.
2
7(i)  0 0  0
     
OC = OB + BC =  0  +  3  =  3 
 h 0  h
     
7(ii) 2OC + OB
By ratio theorem OP =
3
 0   0    0 
1       
=  6  +  0   =  2 
3      
 2h   h    h 
7(iii) Select a suitable direction vector parallel to the plane such as BE = OE − OB
 2h   0   2 h 
     
=  3  −0 =  3  .
 0   h   −h 
     
Thus BE i( a i + b k ) = 0
 2h   a 
   a 1
⇒  3 i 0  = 0 ⇒ =
 −h   b  b 2
  
0 1
  
Since C is on the plane,  3 i 0  = 2h
 h 2
  
1
 
⇒ ri 0  = 2h ⇒ x + 2 z = 2h
 2
 
7(iv) 0 6
   
Given that h = 3 , OP =  2  , OF =  0 
 3 0
   
0 6  0   0
       
n =  2  ×  0  =  18  = 6  3 
 3   0   −12   
       −2 
 0
 
The equation of plane OPF is ri 3  = 0
 −2 
 
1  0 
  
 2 i 3 
 2   −2 
   2
Shortest distance = = units.
02 + 32 + 22 13
8(i) dA 1
= kA − 4000 ⇒ ∫ dA = ∫ 1 dt
dt kA − 4000
1 1
⇒ ∫ dA = ∫ 1 dt
k A − 4000
k
1 4000 4000
⇒ ln A − = t +C ⇒ A− = ekt + kC
k k k
4000
⇒ A− = α ekt ( where α = ± ekC )
k
4000
⇒ A = α ekt +
k
Sub ( t , A ) = ( 0, 60000 ) :
4000 4000
60000 = α + ⇒ α = 60000 − ---- (1)
k k
Sub ( t , A ) = ( 3,69500 ) :
4000
69500 = α e3 k + ---- (2)
k
Sub (1) into (2):
 4000  3k 4000
69500 =  60000 − e +
 k  k
Using G.C., k ≈ 0.1111343 = 0.111 ( 3 s.f.)
 4000  kt 4000
Thus A =  60000 − e +
 k  k
⇒ A = 24000e 0.111t + 36000 with α = 24000 ( 3 s.f.) and λ = 36000 ( 3 s.f.)
8(ii) dA
Since A = π r 2 ,
= 2π r
dr
dr dA dA kA − 4000 kπ r 2 − 4000
Thus = ÷ = =
dt dt dr 2π r 2π r
2
dr ( 0.1111343) π ( 200 ) − 4000
Sub r = 200 , ≈ = 7.93 ( 3 s.f.)
dt 2π ( 200 )
8(iii) Let the rate Mac needs to cut the weeds be n m2 per month.
dA
≈ ( 0.1111343 ) A − n
dt
0 = ( 0.1111343 )( 69500 ) − n ⇒ n = 7720 ( 3 s.f.)
8(iv) dA
= 0 means that the rate which Mac needs to cut the weeds is equal to the rate the
dt
weeds grow. Thus, the area covered in weeds is unchanged.

9(i) 1
2

Amount after 16 days = 1000 ×   = 250 mg


2
9(ii) Amount of I-131 on Day 49
 1
2
  1 2   1 2
=  1000 ×   + 1000  ×   + 1000 ×   + 1000⋯⋯ (*)
  2   2    2 
 1  1  2  1 3 
= 1000 1 + +   +   
 4  4   4  
  1 4 
1 −   
= 1000     = 1328.125 mg = 1328 mg ( nearest mg )
4
 1 
 1− 4 
 
9(iii) 1000
S∞ = = 1333.33 < 1334 mg
1
1−
4
Amount of I-131 will never exceed 1334 mg.
9(iv) Amount of I-125 on Day 121
2
1
= 1000 ×   = 250 mg
2
I-131 is added on Day 17, …, 113, ⇒ total 7 times
Amount of I-131 on Day 121
  1 7 
 1  1 2 1 −   
1  1
6

= 1000 1 + +   + ⋯ +    × = 500     = 666.626 mg


4
 4  4   4   2  1 
 1− 4 
 
Total amount of radioisotopes
= 250 + 666.626 = 917 mg ( nearest mg )

10(a) *
It is not necessarily true because to conclude that i is a root, the coefficients of the
(i) equation must be real.
10(a) 3 2
Sub w = i into z − az + 2az − 4i = 0
(ii)
i3 − ai2 + 2ai − 4i = 0 ⇒ −i + a + 2ai − 4i = 0 ⇒ a(1 + 2i) = 5i ⇒ a = 2 + i
10(a)
(iii)
( 2
Let bz + cz + d ) ( z − i) = 0
By inspection, b = 1 d = 4 ,
( z2 + cz + 4) ( z − i ) = 0
Compare z terms:
4 + 2i − 4
−i c + 4 = 2 a ⇒ c = = −2
−i
Thus z 2 + cz + 4 = 0
2 ± 4 − 4(1)(4) −12
⇒z= = 1± = 1 ± 3i
2 2
⇒ z = 1 + 3i or z = 1 − 3i
10(b) π
arg ( z − 2i ) =
4
π y−2
⇒ tan = ⇒ y = x + 2 where y > 2 , x > 0 --------- (1)
4 x
From z * −1 + i = 2
⇒ x − iy − 1 + i = 2 ⇒ ( x − 1) − i ( y − 1) = 2
2 2
⇒ ( x − 1) + ( y − 1) = 4 --------- (2)
Sub (1) into (2): x 2 − 2 x + 1 + x 2 + 2 x + 1 = 4 ⇒ 2 x2 = 2 ⇒ x = ±1
Since x > 0 , therefore x = 1 ⇒ z = x + iy = 1 + 3i
11(a)
(i)

11(a)
(ii)

11(a)
(iii)

11(b)
(i) Since x = –2 is the asymptote, (–2)2 – b = 0 ⇒ b = 4
 1 1 −8 + a
Substitute the point  −4, −  into G, − = ⇒ a = 5.
 4 4 16 − 4
11(b) y

(ii)

(–1, –1)

x = –2

From G.C, we find the maximum point ( −1, −1) .


For increasing and concave downwards, the only range is −2 < x < −1 .
11(b) 5 x 3 + 2 x 2 − 14 x + 7 6 x + 15
Use long division to obtain = 5x + 2 + 2
(iii) 2
x −4 x −4
6 x + 15 2 x + 5 −1
Thus 5 x + 2 + 2 =0⇒ 2 = (5x + 2)
x −4 x −4 3
−1
Sketch the equation of the line y = ( 5 x + 2 ) onto the diagram to obtain number of
3
intersections = 3. Thus there are 3 distinct real roots to the equation.
3

Section A: Pure Mathematics [40 marks]

1. The position vectors of the fixed points A , B and C relative to the origin O are a , b
and c respectively.
(a) A variable point R has position vector r . Describe the locus of R if
r × (b − a) = 0 . [2]
(b) Given that a = ( 2aib ) b , state the geometrical relation between a and b , and find
b. [3]
(c) A point N divides AB in the ratio 2 :1 and M is the mid-point of BC . Given
that O is the mid-point of MN , show that 2a + 7b + 3c = 0 . [3]

2. It is given that f ( r ) = ( 3r − 2)( 3r + 1) , where r ∈ ℤ + .


n n
1
(i) Use the result
r =1
∑ r2 =
6
n ( n + 1)( 2n + 1) to show that ∑ f ( r ) = n ( an
r =1
2
+ bn + c ) ,

where a , b and c are constants to be determined. [3]

n
1
Let S n = ∑ .
r =1 f ( r )

2
(ii) Show that S2 = , and evaluate S3 and S4 . [2]
7
A
(iii) State a conjecture for Sn in the form , where A and B are in terms of n .
B +1
Prove the conjecture by mathematical induction. [5]

3. The complex number z satisfies the inequalities



z ≤ z + 2 − 2i and < arg ( −2 + 2i − z ) ≤ π .
4
(i) Sketch the locus of z on an Argand diagram. [5]
(ii) Find the exact range of arg ( z + 3) . [3]
(iii) Given that Re ( z ) ≤ 2 , find the area of the region where z can lie in. [2]

4. (i) Find ∫ ln ( 2 x − 1) dx . [3]


(ii) Sketch the curve y = ln ( 2 x − 1) , stating the equations of any asymptotes and the
coordinates of any points where the curve crosses the axes. [2]
(iii) Find the exact area of the region bounded by the curve y = ln ( 2 x − 1) , the x-axis
3 3
and the lines x = and x = . [4]
4 2
(iv) Find the volume of revolution when the region bounded by the curve
3
y = ln ( 2 x − 1) , the x -axis and the lines x = and x = 3 is rotated completely
2
about the y -axis. [3]
4

Section B: Statistics [60 marks]

5. At an international education conference, it is desired to sample 4% of the participants


to seek their opinions about the programs provided at the conference.
(i) Give a reason why it would be difficult to use a stratified sample. [1]
(ii) Explain how a systematic sample could be obtained. [2]

6. (a) Let A and B be events such that


P( A '∩ B) = 0.13 , P( A '∩ B ') = 0.38 and P( A | B) = 0.675 .
(i) Find P( A ∩ B) . [3]
(ii) Determine whether A and B are independent. [1]

(b) Two players, C and D, compete in a match. The probability that C wins the first
set is p. For each set after the first, the conditional probability that any player
wins the set, given that the player won the preceding set, is p. Each set is won
either by player C or D. If there is no limit to the number of sets played, and a
match is won only when a player wins two consecutive sets, show that the
1 − p + p2
probability that C wins the match is .
2− p
[3]

7. Three women and nine men are waiting for a job interview and they are graduates from
Universities A, B and C. The table below shows the number of graduates from each
university.

University Number of
Graduates
A 4 men
B 3 women, 2 men
C 3 men

(a) The twelve graduates sit in a row. Find the number of possible seating
arrangements in which no two women sit next to each other. [2]
(b) The interviewer randomly selects six out of the twelve graduates. Find the
number of possible selections that include graduates from all universities. [3]
(c) The twelve graduates are divided into three groups and each group consists of
four graduates. Find the probability that each group has at most two graduates
from University A. [3]

8. Two independent random variables X and Y are such that X has the distribution
N ( µ1 , 4σ 2 ) and Y has the distribution N ( µ 2 , 9σ 2 ) , where µ1 , µ2 and σ are constants.
µ1
(a) Given that P ( 2 X > Y ) = P ( 9 X < 8Y ) , determine the value of . [5]
µ2
(b) (i) It is given that µ2 = 10 and σ = 1 . Find P (Y < 16 ) . [1]
(ii) One hundred independent observations of Y are taken. Using a suitable
approximation, find the probability that at least 95 of these observations are
less than 16. [3]
5

9. The number of reported cases, x (in hundreds) of a virus in the nth month are given in
the table below.

n 1 2 3 4 5 6
x 0.26 3.05 4.12 4.63 5.15 k

It is given that the least squares regression line of x on n is x = 0.943n + 0.484 .

(i) Show that the value of k to two decimal places is 5.50. [1]

Use the value of k in part (i) for the rest of the question.

(ii) Draw a scatter diagram to illustrate the data. [1]

(iii) Explain why a linear model is inappropriate. [1]

(iv) Explain clearly which of the following models (A) or (B), where a and b are
positive constants, is more appropriate to model these values.

(A) x = a + b ln n ,
(B) x = a + bn2 .

Use the model that you have identified, calculate the values of a and b, and use
them to predict the month in which the number of reported cases is at least 1000.
[2]

(v) Give an interpretation, in context, of the value of a for the model you have
identified in part (iv). [1]

10. On average, 20 customers arrive at John’s restaurant on a day. It is assumed that the
number of customers arriving on a particular day can be modelled by a Poisson
distribution.

(i) Using a suitable approximation, find the probability that the number of customers
arriving in Day 1 and Day 2 differs by at most 2. [4]

(ii) John’s restaurant is opened for 8 hours on a day. The probability that at most two
customers arrive in a period of n hours on a particular day, where n ∈ ℤ+ , is less
than 0.01. Express this information as an inequality in n and hence find the set of
values of n. [4]

(iii) A consultant for John’s restaurant claims that a Poisson distribution may not be a
suitable model for the number of customers arriving on a particular day. Give
two reasons to support the consultant’s claim. [2]
6

11. In a school, the time (in seconds) for a boy to complete a 4 ×10 m shuttle run is denoted
by X. The expected value of X is taken to be 10.8 seconds.

(a) A random sample of 9 boys from the school is chosen and a test at 5%
significance level is carried out to determine if the mean time for a boy to
complete a 4 ×10 m shuttle run has been incorrectly stated. The timings for the 9
boys to complete a 4 ×10 m shuttle run are as shown below.

9.99 10.30 13.01


11.90 10.29 14.12
12.23 12.12 k

It is known that the standard deviation of the above sample is 1.09 seconds.
(i) State the null and alternative hypotheses for the test, defining any symbols
that you used. [1]

(ii) Find the set of values of k for the null hypothesis to be rejected.
State a necessary assumption for the test to be carried out. [5]

(b) A teacher from the school tries a new teaching strategy on a random sample of 50
boys to help them improve on their 4 ×10 m shuttle run timings. The timings, y
(in seconds), for each of these 50 boys to complete a 4 ×10 m shuttle run after the
implementation of the new teaching strategy are summarised by
2
∑ y = 517.49, ∑ ( y − y ) = 122.32.
A test at α % level of significance using this sample results in sufficient evidence
to conclude that the new teaching strategy is effective. Find the minimum value
of α correct to 2 decimal places. State, giving a reason, whether it is necessary to
assume that the population is normally distributed.
[4]

(c) The population standard deviation for the boys in the school to complete a
4 ×10 m shuttle run is now assumed to be 2.8 seconds. A large random sample
of boys of size n is taken from the school. The probability that their mean time to
complete a 4 ×10 m shuttle run exceeds the actual population mean by at least 0.8
seconds is not more than 0.001. Form an inequality involving n and hence find
the least possible value of n. [4]
2015 H2 Mathematics Prelim Paper 2 Solutions

Qn Solutions
1. Locus of R is a straight line passing through the origin and parallel to AB .
(a)

1. a and b are parallel


(b) a = ( 2aib ) b
2
a = 2 a b cos θ
1
b=
2

1. Using ratio theorem,


(c) 1 2 1
ON = a + b and OM = ( b + c )
3 3 2
Since O is the midpoint of MN,
ON = −OM
1 2 1
a + b = − (b + c)
3 3 2
2a + 4b = −3b − 3c
2a + 7b + 3c = 0

2. f ( r ) = ( 3r − 2 )( 3r + 1) = 9r 2 − 3r − 2
(i) n n

∑ f ( r ) = ∑ ( 9r 2 − 3r − 2 )
r =1 r =1
n n n
= 9∑ r 2 − 3∑ r − ∑ 2
r =1 r =1 r =1

3 3
= n ( n + 1)( 2n + 1) − n ( n + 1) − 2n
2 2
3 2
= 3n + 3n − 2n
= n ( 3n 2 + 3n − 2 )
Hence, a = 3 , b = 3 and c = −2 .

2. 1 1
(ii) S1 = =
( 3 − 2 )( 3 + 1) 4
1 1 2
S2 = + = ( qed )
4 ( 6 − 2 )( 6 + 1) 7
2 1 3
S3 = + =
7 ( 9 − 2 )( 9 + 1) 10
3 1 4
S4 = + =
10 (12 − 2 )(12 + 1) 13

1
Qn Solutions
2. n
(iii) Conjecture: Sn =
3n + 1
n
Let P ( n ) be the proposition that “ Sn = for all n ∈ ℤ + ”.
3n + 1
1
Consider P (1) , LHS = S1 = = RHS .
4
Therefore P (1) is true.
k
Assume P ( k ) is true for some k ∈ ℤ + , i.e., Sk = .
3k + 1
k +1
Want to show P ( k + 1) is true, i.e., Sk +1 = .
3 ( k + 1) + 1
Consider P ( k + 1) ,
k +1
1
S k +1 = ∑
r =1 f ( r )

k 1
= +
3k + 1 ( 3 ( k + 1) − 2 ) ( 3 ( k + 1) + 1)
k ( 3k + 4 ) + 1
=
( 3k + 1)( 3k + 4 )
=
( 3k + 1)( k + 1)
( 3k + 1)( 3k + 4 )
k +1
=
3 ( k + 1) + 1
∴ P ( k + 1) is true.
Since P (1) is true and P ( k ) is true ⇒ P ( k + 1) is true, by mathematical induction,
+
P ( n ) is true for all n ∈ ℤ .
3. 3π
(i) < arg ( −2 + 2i − z ) ≤ π
4

< arg ( −1) + arg ( z + 2 − 2i ) ≤ π
4
π
− < arg ( z − ( −2 + 2i ) ) ≤ 0 -----(*)
4

2
Qn Solutions
Im

Re
O

3. Im
(ii)

max arg
Re
O
min arg

2
max arg ( z + 3) = angle of ( 0, 2 ) from ( −3, 0 ) = tan −1  
3
π 2
− < arg ( z + 3) ≤ tan −1  
4 3
3. Im
(iii)

A
B
Re
O

Area = triangle A + trapezium B


1 1
= (1)( 2 ) + ( 2 )( 2 + 4 ) = 7
2 2

3
Qn Solutions
4.
(i)
∫ ln ( 2 x − 1) dx
2
= x ln ( 2 x − 1) − ∫ x dx
2x −1
1
= x ln ( 2 x − 1) − ∫ 1 + dx
2x −1
1
= x ln ( 2 x − 1) − x − ln ( 2 x − 1) + c
2
 1
=  x −  ln ( 2 x − 1) − x + c
 2
4.
y
(ii)

4.
(iii) y

1 x

Area of shaded region


1 3/2
= ∫ − ln( 2 x − 1)dx + ∫ ln( 2 x − 1)dx
3/4 1
1
 1 
= −  x ln( 2 x − 1) − x − ln( 2 x − 1) 
 2  3/4
3/2
 1 
+  x ln( 2 x − 1) − x − ln( 2 x − 1) 
 2 1

4
Qn Solutions
  3 1 3 1 1 
= − ( 0 − 1 − 0 ) −  ln − − ln  
  4 2 4 2 2 
 3 3 1  
+  ln 2 − − ln 2  − ( 0 − 1 − 0 ) 
 2 2 2  
3 1
= ln 2 − square units
4 4
4.
(iv)

2
3 ln 5
Volume = π (3) 2 (ln 5) − π   ln 2 − π ∫ x 2 dy
2 ln 2

2 2
3 ln 5  e + 1 
y
= π (3) (ln 5) − π   ln 2 − π ∫ 
2
 dy
2 ln 2
 2 

= 26.927 = 26.9 cubic units (3s.f.)


5(i) In order to use stratified sampling, we need to know the complete composition of
the participants according to strata, e.g. race, gender or age-groups. However, it is
difficult to have this complete information due to absentees, incomplete registration
information, etc. Hence it would be difficult to use a stratified sample.

5(ii) We assign an index number to each participant when they arrive at the conference.
Let the total number of participants to arrive at the conference be x. Sampling
x
interval = = 25 . Pick a random number from 1 to 25, then pick every 25th
0.04 x
participant thereafter. E.g. If the number 5 was selected, we sample the participants
with numbers 5, 30, 55, … until 4% of the participants are sampled.

6 P( A ∩ B )
(ai) P( A | B) = 0.675 =
P( B)
A B
⇒ P( A ∩ B) = 0.675 ( P( A ∩ B) + 0.13)
⇒ P( A ∩ B) = 0.675P( A ∩ B) + 0.08775 0.13
⇒ P( A ∩ B) = 0.27

5
Qn Solutions

Alternative 1
P ( A ∪ B ) = P ( A) + P ( B ) − P ( A ∩ B )
P ( A ∩ B)
0.62 = 0.49 + − P ( A ∩ B)
P ( A | B)
⇒ P ( A ∩ B ) = 0.27

Alternative 2
P ( A '∩ B )
P ( A' | B) = ⇒ P ( B ) = 0.4
P (B)
P ( A ∩ B ) = P ( B ) P ( A | B ) = 0.27

6 P( A | B) = 0.675
(aii) P( A) = 1 − 0.13 − 0.38 = 0.49
P( A | B) ≠ P( A)
Therefore the two events A and B are not independent

Or

P( A ∩ B) = 0.27
P( A) × P( B) = 0.49 × 0.4 = 0.196
P( A ∩ B) ≠ P( A) × P( B )
Therefore the two events A and B are not independent

6 Required Probability
(b) P(CC ) + P(CDCC ) + P(CDCDCC ) + ...
=
+ P( DCC ) + P( DCDCC ) + P( DCDCDCC ) + ...
2 4
p 2 + p 2 (1 − p ) + p 2 (1 − p ) + ....
= 2 4 6
+ (1 − p ) p + (1 − p ) p + (1 − p ) p + ....
 1  2
 1 
= p2  2  + (1 − p ) p  2
1 − (1 − p )  1 − (1 − p ) 
p  p + (1 − p ) 
2

=  
2
2p − p
1 − p + p2
=
2− p
(shown)
7(a) 9!× 10C3 × 3! = 261273600

6
Qn Solutions
7(b) [All Universities]=[Any six-person]
– [University A&B]
– [University B&C]
– [University A&B]

9
Case 1: University A&B C6
8
Case 2: University B&C C6
7
Case 3: University A&C C6
So the answer is 12C6 − 9C6 − 8C6 − 7C6 = 805

Alternate Method [Not advisable because of too many cases]


Consider number of candidates from each university.

If we have x from A, y from B and z from C, the number of possible ways is


4
C x 5C y 3C z

A(4) 1 1 1 2 2 2 3 3 4
B(5) 2 3 4 1 2 3 1 2 1
C(3) 3 2 1 3 2 1 2 1 1
Ans 40 120 60 30 180 180 60 120 15

So the total number of possible way is 805.


12
7(c) C4 8C4 4 C4
All possible groupings: = 5775
3!
Method 1 (Direct Method)
Case 1: Candidates from University A grouped in 2,2,0
(4 from A versus 8 from ‘the rest’)
Number of ways = 4C2 8C2 × 2C2 6C2 ÷ 2! = 1260

Case 2: Candidates from University A grouped in 2,1,1


Number of ways = 4C2 8C2 × 2C1 6C3 ÷ 2! = 3360
1260 + 3360
So the probability is = 0.8
5775

Method 2 (Method of Complementation –cases where a group has more than 2


candidates from University A)
Case 1: Candidates from University A grouped in 3,1,0
Number of ways = 4C3 8C1 × 8C4 ÷ 2! = 1120
Case 2: Candidates from University A grouped in 4,0,0
Number of ways = 4C4 × 8C4 ÷ 2! = 35
1120 + 35
So the probability is 1− = 0.8
5775

7
8(a)
(
2 X − Y ∼ N 2µ1 − µ 2 , 22 ( 4σ 2 ) + 9σ 2 )
i.e. 2 X − Y ∼ N ( 2µ1 − µ2 , 25σ 2 )

(
9 X − 8Y ∼ N 9µ1 − 8µ 2 ,92 ( 4σ 2 ) + 82 ( 9σ 2 ) )
i.e. 9 X − 8Y ∼ N ( 9µ1 − 8µ 2 ,900σ 2 )
P ( 2 X > Y ) = P ( 9 X < 8Y )
P ( 2 X − Y > 0 ) = P ( 9 X − 8Y < 0 )
 ( 2 X − Y ) − ( 2 µ1 − µ2 ) 0 − ( 2 µ1 − µ2 ) 
⇒ P > 
2
 25 σ 25σ 2 
 ( 9 X − 8Y ) − ( 9 µ1 − 8µ2 ) 0 − ( 9 µ1 − 8µ2 ) 
= P > 
 900σ 2 900σ 2 
 0 − ( 2 µ1 − µ 2 )   0 − ( 9 µ1 − 8µ2 ) 
⇒ P Z >  = P Z < 
 25σ 2   900σ 2 
 µ − 2 µ1   8µ 2 − 9 µ1 
⇒ P Z > 2  = P Z < 
 5σ   30σ 

b a

By symmetry of the standard normal distribution,


µ − 2 µ1 8µ − 9 µ1
So 2 =− 2
5σ 30σ
⇒ 6µ2 − 12µ1 = −8µ2 + 9µ1
⇒ 14 µ2 = 21µ1
µ 14 2
∴ 1 = =
µ 2 21 3
8 Y ∼ N (10,9 )
(bi)
P (Y < 16 ) = 0.97725 = 0.977 (3 sf)

8 Let S be the number of observations with Y ≥ 16


(bii) Hence S ∼ B (100, 0.02275 )
Since n = 100 is sufficiently large, np = 2.275 < 5 ,
S ∼ Po ( 2.275 ) approximately
∴ P (100 − S ≥ 95 ) = P ( S ≤ 5 ) = 0.971 (3 s.f.)

8
9(i) 17.21 + k
x= , n = 3.5
6
( n , x ) lies on the regression line, so
x = 0.943n + 0.484
17.21 + k
= 0.943 ( 3.5 ) + 0.484
6
k = 5.50 ( 2 dp )
9(ii)
x ( 6,5.50 )

9 From the scatter plot, x and n have a curvilinear relationship. Therefore a linear
(iii) model is inappropriate even though the product moment correlation coefficient is
relatively high (i.e. 0.915).

9
(iv) The graph of x = a + bn2 is concave upwards (or increase at increasing rate) similar
to the scatter plot

The graph of x = a + b ln n is concave downwards (or increase at decreasing rate) .

From GC,
rB = 0.8212061688 ≈ 0.821( 3 s.f.)
rA = 0.9859197289 ≈ 0.986 ( 3 s.f.) which is closer to 1 and hence suggested a
relatively stronger linear relationship between x and ln n as compared to x and n2 .
Therefore Model (A) is more appropriate.

9
9 Using GC, the regression line of x on ln n is
(iv) x = 0.638570346 + 2.869411323ln n
b = 2.869411323 ≈ 2.87
a = 0.638570346 ≈ 0.639
For x ≥ 10,
2.869411323ln n + 0.638570346 ≥ 10
ln n ≥ 3.262491362
n ≥ 26.1

The number of reported cases is at least 1,000 in the 27th month


9 The value of a represents the estimated number of cases (in hundreds) of the virus
(v) reported in the first month.

10 Let X 1 , X 2 be the number of customers on Day 1 & Day 2 respectively


(i)
Then X 1 ∼ Po ( 20 ) , X 2 ∼ Po ( 20 )
Since λ = 20 > 10,
X 1 ∼ N ( 20, 20 ) approximately,
X 2 ∼ N ( 20, 20 ) approximately.
So X 1 − X 2 ∼ N ( 0, 40 )
P ( X 1 − X 2 ≤ 2 ) = P ( −2 ≤ X 1 − X 2 ≤ 2 )
c .c .
 → P ( −2.5 ≤ X 1 − X 2 ≤ 2.5 ) = 0.307
10 Let Y be the number of customers arrived in n hours
(ii)  20 
So, Y ∼ Po  n 
 8 
i.e. Y ∼ Po ( 2.5n ) where n ∈ ℤ + , 0 ≤ n ≤ 8 .
P (Y ≤ 2 ) < 0.01
⇒ P (Y = 0 ) + P (Y = 1) + P ( Y = 2 ) < 0.01
 ( 2.5n )0 ( 2.5n )1 ( 2.5n )2 
⇒e 
−2.5 n
+ +  < 0.01
 0! 1! 2! 
1 + 2.5n + 3.125n 2
⇒ < 0.01
e2.5 n
When n = 3, P (Y ≤ 2 ) = 0.02026 > 0.01
When n = 4, P (Y ≤ 2 ) = 0.00277 < 0.01
So the set of values of n is {4,5, 6, 7,8} or {n ∈ ℤ + : 4 ≤ n ≤ 8}
10 1. The arrival of customers may not be independent, for example, relatives/friends
(iii) visit the restaurant together.

2. The mean number of customers per unit time may not be constant throughout the
day. For example, lunch time we may expect more customers.

10
11 Let µ be the population mean time (in seconds) for a school boy in the school to
(ai) complete a 4 × 10 m shuttle run.

H0 : µ = 10.8
H1 : µ ≠ 10.8

11 Assumption: Since sample size is small and population variance is unknown, we


(aii) need to assume that the time, in seconds, for a school boy to complete a 4 × 10 m
shuttle run follows a normal distribution.

x=
∑ x = 93.96 + k
9 9
Sample variance = 1.092
9 2
s 2 = (1.09 ) = 1.3366125
8

Under H0, Test Statistic


X − 10.8
T= ∼t (8)
1.3366125
9
x − 10.8
Test Statistic value, t =
1.3366125
9

Reject H0 at 5% level of significance,


P ( T ≥ t ) ≤ 0.05

x − 10.8
≥ 2.306004133
1.3366125
9

 93.96 + k 
  − 10.8
 9  ≤ −2.306004133
1.3366125
9
k ≤ −4.758049639
(rejected since k > 0)

11
 93.96 + k 
  − 10.8
or  9  ≥ 2.306004133
1.3366125
9
k ≥ 11.2 (3 sf)

{k : k ∈ ℝ , k ≥ 11.2}
11
y=
∑ y = 517.49
(b)
The unbiased estimate for population mean is y 50 50
51749
y= = 10.3498
5000

The unbiased estimate for population variance is s 2


2
2 ∑( y − y)
s =
n −1
122.32 3058
s2 = = = 2.496326531
49 1225

H0 : µ = 10.8
H1 : µ < 10.8

Under H0, Test Statistic


X − 10.8
Z= ∼ N ( 0,1) approximately
 3058 
 
 1225 
50
by Central Limit Theorem since n (=50) is large

From G.C., p-value = 0.0219608 ≈ 0.0220 (3 s.f.)


Reject H0, α% ≥ 2.20% and thus α = 2.20 .

Since n (=50) is large, the mean timing for the boys to complete a 4 × 10 m shuttle
run will be approximated to a normal distribution by Central Limit Theorem.
Therefore not necessary to assume population follows normal distribution.
11  2.82 
(c) Given n is large, by Central Limit Theorem, X ∼ N  µ , .
 n 

P( X − µ ≥ 0.8) ≤ 0.001
 
 0.8 
PZ ≤  ≥ 0.999
 2.8 
 
 n

12
0.8
≥ 3.090232308
2.8
n
n ≥ 116.98
∴ n = 117

13

You might also like